$\require{enclose}$ $\newcommand{\avsum}{\mathrel{\displaystyle\int \!\!\!\!\!\! \Delta\ }}$ $\newcommand{\bcancelto}[2]{{\enclose{southeastarrow}{#2}\,}_{\lower.75ex{#1}}}$ $\newcommand{\ordcirc}[1]{\mathrel{[\hspace{-4pt} \circ \hspace{2pt}#1 \hspace{3pt}]\hspace{-4pt}\circ}}$ $\newcommand{\avigual}{\{=\}}$ $\newcommand{\intsup}{{\LARGE \big\uparrow}\displaystyle\int}$ $\newcommand{\intinf}{{\LARGE \big\downarrow}\displaystyle\int}$
Última atualização estrutural do weblog: 18-05-2024.

Este weblog utiliza serviços de terceiros, e os mesmos podem não funcionar adequadamente, o que não depende de mim.

Se as expressões matemáticas não estiverem satisfatoriamente visíveis, você pode alterar as configurações de exibição no menu contextual.

Este weblog pode passar por melhorias. Caso não teve uma boa experiência hoje, futuramente os problemas poderão estar corrigidos.

Em caso de não ser a mim mais possível realizar manutenções, como, por exemplo, devido a falecimento ou desaparecimento, alguns links podem ficar quebrados e eu não responder mais a comentários. Peço compreensão.
Mostrando postagens com marcador integrais. Mostrar todas as postagens
Mostrando postagens com marcador integrais. Mostrar todas as postagens

sexta-feira, 17 de dezembro de 2021

Calcular $I\ =\ \displaystyle\int_0^a \sqrt{a^2 - x^2}\ dx$, $a > 0$.

$I = a\displaystyle\int_0^a \sqrt{1 - \left(\dfrac{x}{a}\right)^2}\ dx$

Seja $y = \dfrac{x}{a}$. $dy = \dfrac{dx}{a}$

$I = a^2\displaystyle\int_0^1 \sqrt{1 - y^2}\ dy$

Seja $y = \sin \theta$, $-\dfrac{\pi}{2} \le \theta \le \dfrac{\pi}{2}$. $dy = \cos \theta\ d\theta$.

$I = a^2\displaystyle\int_0^{\pi / 2} \cos^2 \theta\ d\theta\ =\ a^2\displaystyle\int_0^{\pi / 2} \dfrac{(\cos 2\theta) + 1}{2} d\theta\ =\ a^2\displaystyle\int_0^{\pi / 2} \cos 2\theta\ d\theta + \dfrac{a^2\pi}{4}$

Seja $\varphi = 2\theta$. $d\varphi = 2 d\theta$.

$I = \cancelto{0}{\dfrac{a^2}{2}\displaystyle\int_0^{\pi} \cos \varphi\ d\varphi} + \dfrac{a^2\pi}{4}\ =\ \fbox{$\dfrac{a^2\pi}{4}$}$

domingo, 12 de dezembro de 2021

Por meio da integração, encontrar o volume do cone de raio da base $r = 2$ e altura $h = 5$.

O cone será resultante da rotação da reta $y = \dfrac{rx}{h}$, para $x \in [0, h]$, união o círculo $y^2 + z^2 \le r^2\ \wedge\ x = h$.

Tal volume será dado por $\pi \cdot \dfrac{r^2}{h^2}\displaystyle\int_0^5 x^2\ dx = \pi \cdot \dfrac{4}{25} \cdot \left. \dfrac{x^3}{3}\right|_0^5 = \fbox{$\dfrac{20\pi}{3}$}$

sexta-feira, 10 de dezembro de 2021

Encontrar a área delimitada pelo eixo $Ox$ e a parábola $y = x^2 + x - 2$.

$\left|\displaystyle\int_{-2}^1 x^2 + x - 2\ dx\right| = \left| \left. \left(\dfrac{x^3}{3} + \dfrac{x^2}{2} - 2x\right)\right|_{-2}^1 \right| = \left| \dfrac{1}{3} + \dfrac{1}{2} - 2 + \dfrac{8}{3} - 2 - 4 \right| =$

$= \left| -\dfrac{9}{2}\right| = \fbox{$\dfrac{9}{2}$}$

Determinar a soma de Riemann para $f(x) = 2 - x^2$, e $P$ a partição de $[0, 2]$ em 4 subintervalos de mesmo comprimento, escolhendo $c_i$ como sendo o extremo direito do subintervalo $[x_{i-1}, x_i]$.

$S_4(f) = \displaystyle\sum_{i=1}^4 f(c_i)(x_i - x_{i-1}) = \dfrac{1}{2}\displaystyle\sum_{i=1}^4 f(x_i) =$

$= \dfrac{1}{2}\left[\left(2 - \dfrac{1}{4}\right) + (2 - 1) + \left(2 - \dfrac{9}{4}\right) + (2 - 4)\right] = \fbox{$\dfrac{1}{4}$}$

sábado, 23 de outubro de 2021

$\displaystyle\int \dfrac{dx}{x^4 + 1}$.

$\dfrac{1}{x^4 + 1} = \dfrac{1}{(x^2 - \sqrt{2}x + 1)(x^2 + \sqrt{2}x + 1)} =$

$= \dfrac{1}{4} \cdot \dfrac{-\sqrt{2}x + 2}{x^2 - \sqrt{2}x + 1} + \dfrac{1}{4} \cdot \dfrac{\sqrt{2}x + 2}{x^2 + \sqrt{2}x + 1}$

$\displaystyle\int \dfrac{dx}{x^4 + 1} = \dfrac{-1}{2} \displaystyle\int \dfrac{\sqrt{2}x - 2}{(\sqrt{2}x - 1)^2 + 1}\ dx\ + \dfrac{1}{2} \displaystyle\int \dfrac{\sqrt{2}x + 2}{(\sqrt{2}x + 1)^2 + 1}\ dx\ =$

 

$= \dfrac{-\sqrt{2}}{4} \displaystyle\int \dfrac{u - 1}{u^2 + 1}\ du + \dfrac{\sqrt{2}}{4} \displaystyle\int \dfrac{v + 1}{v^2 + 1}\ dv =$


$= \scriptsize{\fbox{$\dfrac{-\sqrt{2}}{8} \log [(\sqrt{2}x - 1)^2 + 1] + \dfrac{\sqrt{2}}{8} \log [(\sqrt{2}x + 1)^2 + 1] + \dfrac{\sqrt{2}}{4} \arctan (\sqrt{2}x - 1) + \dfrac{\sqrt{2}}{4} \arctan (\sqrt{2}x + 1) + c$}}$

quarta-feira, 30 de junho de 2021

A inversa, a derivada, e a integral da função corda.


${\large cord\ \alpha = \sqrt{2(1 - \cos \alpha)}}$

Inversa: seja $arccord: \underset{x\ \mapsto\ arccord\ x}{[0, 2] \rightarrow [0, \pi]},\ \fbox{$arccord\ x = \arccos \left(1 - \dfrac{x^2}{2}\right)$}$.



Derivada: $\fbox{$(cord\ \alpha)' = \dfrac{\sin \alpha}{\sqrt{2 - 2\cos \alpha}}$}$.



Observemos que, para $0 \le \alpha \le 2\pi$, $cord\ \alpha = 2\sin \dfrac{\alpha}{2}$.

Logo,

$\fbox{$\displaystyle\int cord\ \alpha\ d\alpha\ =\ -4\cos \dfrac{\beta}{2} + c,\ \alpha = 2k\pi + \beta,\ k \in \mathbb{Z}, 0 \le \beta < 2\pi$}$.

terça-feira, 29 de junho de 2021

Seja $V$ o espaço vetorial de dimensão infinita gerado por $\{\sin \alpha x\ :\ \alpha \in \mathbb{Z}\}$ e $\langle f, g \rangle = \displaystyle\int_{-\pi}^{\pi} f(x)g(x)\ dx$, mostre que $\sin mx$ e $\sin nx$, com $m, n \in \mathbb{Z},\ m \neq n$ são linearmente independentes.

Seja $V$ o espaço vetorial de dimensão infinita gerado por $\{\sin \alpha x\ :\ \alpha \in \mathbb{Z}\}$ e $\langle f, g \rangle = \displaystyle\int_{-\pi}^{\pi} f(x)g(x)\ dx$, mostre que $\sin mx$ e $\sin nx$, com $m, n \in \mathbb{Z},\ m \neq n$ são linearmente independentes.

Resolução:

Basta mostrar que $\sin mx$ e $\sin nx$, com $m, n \in \mathbb{Z},\ m \neq n$ são perpendiculares.

De fato, $\displaystyle\int_{-\pi}^{\pi} (\sin mx)(\sin nx)\ dx\ =\ \dfrac{1}{2}\displaystyle\int_{-\pi}^{\pi} \cos (m - n)x - \cos (m + n)x\ dx\ =\ 0$.

Quod Erat Demonstrandum.

terça-feira, 22 de junho de 2021

Área sob uma parábola com concavidade para baixo dadas as intersecções com $Ox$ e a ordenada do vértice.

Sejam $P(x)$ a parábola em questão, $a$ e $b$, $b > a$ as intersecções com $Ox$, e $y_V$ a ordenada do vértice de $[-x^2 + (a+b)x - ab]$, e $h$ a ordenada do vértice de $P(x)$, $h > 0$.

$P(x) = \dfrac{h}{y_V}[-x^2 + (a+b)x - ab]$, é a equação cartesiana de tal parábola.

$y_V = \dfrac{\Delta}{4} = \dfrac{(a+b)^2 - 4ab}{4}$

Logo $P(x) = \dfrac{4h}{(a+b)^2 - 4ab}[-x^2 + (a+b)x - ab]$.

Logo a área $A$ será $A = \dfrac{4h}{(a+b)^2 - 4ab} \displaystyle\int_a^b -x^2 + (a+b)x - ab\ dx = \dfrac{4h}{(a+b)^2 - 4ab} \left. [-\dfrac{x^3}{3} + \dfrac{(a+b)x^2}{2} - abx] \right|_a^b$.

$\fbox{$A = \dfrac{4h}{(a+b)^2 - 4ab}[-\dfrac{b^3}{3} + \dfrac{(a+b)b^2}{2} - ab^2 + \dfrac{a^3}{3} - \dfrac{(a+b)a^2}{2} + a^2b]$}$

Exemplo:

Sejam $a = 0$, $b = 1$, e $h = 1$:



$A = 4(-\dfrac{1}{3} + \dfrac{1}{2}) = \dfrac{2}{3}$.

Decomposição de frações com binômios no denominador em frações parciais.

Sejam $p, r \in \mathbb{R}[x]$, $a \in \mathbb{R}$ com $r(a) \neq 0$ e $n \in \mathbb{N}$. Então existem $B \in \mathbb{R}$ e $q \in \mathbb{R}[x]$ tais que

$\dfrac{p(x)}{r(x)(x - a)^n} = \dfrac{q(x)}{r(x)(x - a)^{n-1}} + \dfrac{B}{(x - a)^n}$.

Resolução:

Basta mostrar que $p(x) = q(x)(x - a) + Br(x)$.

Definamos $B = \dfrac{p(a)}{r(a)}$. Definamos também $h(x) = p(x) - Br(x)$.

Obviamente $h(a) = 0$, logo, por D'Alembert, $h(x) = q(x)(x - a)$.

Logo $p(x) = q(x)(x - a) + Br(x)$.

C.Q.D.

Região entre os gráficos de $y = x^3$ e $y = -2x^2$.

Esboce a região finita $R$ entre os gráficos de $y = x^3$ e $y = -2x^2$.

a) Calcule a área da região $R$.

b) Determine o volume do sólido $E$ obtido com a rotação da região $R$ em torno da reta $y = 2$.

Resolução:



Inicialmente vamos encontrar as intersecções entre os gráficos.

$x^3 + 2x^2 = 0\ \Rightarrow\ \text{Os pontos são}\ (0, 0)\ \text{e}\ (-2, -8)$

A área é $\left| \displaystyle\int_{-2}^0 (x^3 + 2x^2)\ dx \right|  = \left| \left. (\dfrac{x^4}{4} + \dfrac{2x^3}{3})\right|_{-2}^0 \right| = \fbox{$\dfrac{4}{3}$}$

O volume procurado é $\pi \left| \displaystyle\int_{-2}^0 [(x^3 - 2)^2 - (-2x^2 - 2)^2]\ dx \right| = \pi \left| \left. (\dfrac{x^7}{7} - x^4 + 4x - \dfrac{4x^5}{5} - \dfrac{8x^3}{3} - 4x) \right|_{-2}^0 \right| = \fbox{$\dfrac{1328\pi}{105}$}$


Determinar o volume de $4$ nódulos resultantes da rotação da função $y = \sin x$ em torno do eixo $x$.

Determinar o volume de $4$ nódulos resultantes da rotação da função $y = \sin x$ em torno do eixo $x$.

Resolução:



$V = 4\pi \displaystyle\int_0^{\pi} \sin^2 x\ dx = 4 \displaystyle\int_0^{\pi} \dfrac{1 - \cos 2x}{2}\ dx = \left. [2x - \sin(2x)] \right|_0^{\pi} = \fbox{$2\pi$}$


Calcular $I = \displaystyle\int x^2 e^x \cos x\ dx$.

Calculemos inicialmente $J = \displaystyle\int e^x \cos x\ dx$.

Aplicando "por partes":

$J = e^x \cos x + \displaystyle\int e^x \sin x\ dx = e^x \cos x + e^x \sin x - \underset{J}{\underbrace{\displaystyle\int e^x \cos x\ dx}}$

$2J = e^x(\cos x + \sin x)\ \Rightarrow\ J = \dfrac{e^x(\cos x + \sin x)}{2} + c_1$

Calculemos também inicialmente $K = \displaystyle\int e^x \sin x\ dx$.

Aplicando "por partes":

$K = e^x \sin x - \displaystyle\int e^x \cos x\ dx = e^x \sin x - e^x \cos x - \underset{K}{\underbrace{\displaystyle\int e^x \sin x\ dx}}$

$2K = e^x(\sin x - \cos x)\ \Rightarrow\ K = \dfrac{e^x(\sin x - \cos x)}{2} + c_2$

Também é útil saber $L = \displaystyle\int J\ dx$:

$L = \dfrac{J + K}{2} + c_1x$

Também é útil saber $M = \displaystyle\int K\ dx$:

$M = \dfrac{K - J}{2} + c_2x$

Continuando, aplicando "por partes" em $I$:

$I = x^2\dfrac{e^x(\cos x + \sin x)}{2} - \displaystyle\int xe^x(\cos x + \sin x)\ dx = x^2\dfrac{e^x(\cos x + \sin x)}{2} -$

$- xJ - xK + \underset{L + M}{\underbrace{\displaystyle\int J + K\ dx}}$

Logo $\fbox{$\displaystyle\int x^2 e^x \cos x\ dx = x^2\dfrac{e^x(\cos x + \sin x)}{2} - xe^x \sin x + \dfrac{e^x(\sin x - \cos x)}{2} + c$}$.

Calcular $I = \displaystyle\int (\sin x) \log (\cos x)\ dx$.

Seja $u = \cos x$, com $x \in [0, \dfrac{\pi}{2}[$, $du = -\sin x\ dx$.

$I = -\displaystyle\ \log u\ du$

Aplicando "por partes":

$I = -u\log u + \displaystyle\int \dfrac{u}{u} du = c + u - u\log u = \fbox{$\cos x - (\cos x)(\log \cos x) + c$}$

Encontrar o volume do sólido de revolução gerado pela rotação de $y = \dfrac{1}{x}$ em torno do eixo $x$, com $x \in [1, 5]$.

Encontrar o volume do sólido de revolução gerado pela rotação de $y = \dfrac{1}{x}$ em torno do eixo $x$, com $x \in [1, 5]$.

Resolução:



O volume será dado por $V = \pi \displaystyle\int_1^5 \dfrac{dx}{x^2}$.

$V = \left. -\dfrac{\pi}{x} \right|_1^5 = -\dfrac{\pi}{5} + \pi = \fbox{$\dfrac{4\pi}{5}$}$


Mostre que o Primeiro Teorema Fundamental do Cálculo e o Segundo Teorema Fundamental do Cálculo são equivalentes.

Pelo Primeiro Teorema Fundamental do Cálculo, se $F$ é uma primitiva de $f$, então $F'(x) = f(x)$.

$[F(x) - \underset{\text{Constante}}{\underbrace{F(a)}}]' = f(x)$

$\dfrac{d}{dx}[\displaystyle\int_a^x f(t)\ dt] = f(x)$

$\fbox{$\displaystyle\int_a^x f(t)\ dt = F(x)$}$, que é o Segundo Teorema Fundamental do Cálculo.

C.Q.D.

Segundo Teorema Fundamental do Cálculo.

Seja $f:\ \mathbb{R} \rightarrow \mathbb{R}$ e considere a função $g(x) = \displaystyle\int_a^x f(t)\ dt$. Mostre que $g'(x) = f(x)$.

Resolução:

Seja $F$ uma primitiva de $f$.

$g(x) = \displaystyle\int_a^x f(t)\ dt = F(x) - \underset{\text{Constante}}{\underbrace{F(a)}}$

Derivando ambos os membros com relação a $x$:

$g'(x) = f(x)$

C.Q.D.

Seja $f:\ \mathbb{R} \rightarrow [a, +\infty[$, contínua e suponha que $\displaystyle\lim_{x \rightarrow +\infty} f(x) = L$. Prove que $\displaystyle\lim_{b \rightarrow +\infty} \dfrac{1}{b-a}\displaystyle\int_a^b f(x)\ dx = L$.

Seja $f:\ \mathbb{R} \rightarrow [a, +\infty[$, contínua e suponha que $\displaystyle\lim_{x \rightarrow +\infty} f(x) = L$. Prove que

$\displaystyle\lim_{b \rightarrow +\infty} \dfrac{1}{b-a}\displaystyle\int_a^b f(x)\ dx = L$.

Resolução:

Seja $F$ uma primitiva de $f$.

$\displaystyle\lim_{b \rightarrow +\infty} \dfrac{1}{b-a}\displaystyle\int_a^b f(x)\ dx = \displaystyle\lim_{b \rightarrow +\infty} \dfrac{F(b) - F(a)}{b - a}$, que chamaremos de $Q$.

$\displaystyle\lim_{b \rightarrow +\infty} \dfrac{F(b) - F(a)}{b - a} = Q$

$\displaystyle\lim_{a \rightarrow b} \displaystyle\lim_{b \rightarrow +\infty} \dfrac{F(b) - F(a)}{b - a} = \displaystyle\lim_{a \rightarrow b} Q$

$\displaystyle\lim_{b \rightarrow +\infty} \displaystyle\lim_{a \rightarrow b} \dfrac{F(b) - F(a)}{b - a} = Q$

$\displaystyle\lim_{b \rightarrow +\infty} f(b) = Q$

$L = Q$

C.Q.D.

Teorema do Valor Médio para Integrais.

Mostre que se $f:\ [a, b]\ \rightarrow\ \mathbb{R}$ é contínua, então existe $c \in [a, b]$ tal que

$\displaystyle\int_a^b f(x)\ dx = f(c)(b - a)$.

Resolução:

O Teorema do Valor Médio afirma: seja  $g$ uma função contínua em $[a, b]$ e derivável em $(a, b)$, então, existe $c \in (a, b)$ tal que

$g'(c) \cdot (b - a) = g(b) - g(a)$.

$g'(c) \cdot (b - a) = \displaystyle\int_a^b g'(x)\ dx$

Seja $f(x) = g'(x)$:

$\fbox{$\displaystyle\int_a^b f(x)\ dx = (b - a)f(c)$}$

C.Q.D.

segunda-feira, 21 de junho de 2021

Valor de uma função dadas certas condições e uma integral.

Seja $f:\ \mathbb{R} \rightarrow \mathbb{R}$ uma função duas vezes continuamente diferenciável, isto é, $f$, $f'$ e $f''$ são contínuas. Determine o valor de $f(0)$ sabendo que $f(\pi) = 2$ e que

$\displaystyle\int_0^{\pi} (f(x) + f''(x))\sin x\ dx\ =\ 5$.

Resolução:

Seja $I = \displaystyle\int (f(x) + f''(x))\sin x\ dx$.

$I = \displaystyle\int f(x) \cdot \sin x\ dx\ +\ \displaystyle\int f''(x) \cdot \sin x\ dx\ =$

$= -f(x) \cdot \cos x + \cancel{\displaystyle\int f'(x) \cdot \cos x\ dx}\ +\ f'(x) \cdot \sin x\ \cancel{- \displaystyle\int f'(x) \cdot \cos x\ dx}\ =$

$= -f(x) \cdot \cos x + f'(x) \cdot \sin x\ + c$

Logo $\displaystyle\int_0^{\pi} (f(x) + f''(x))\sin x\ dx\ =\ \left. (-f(x) \cdot \cos x + f'(x) \cdot \sin x)\ \right|_0^{\pi}\ =\ 2 + f(0)$.

$\displaystyle\int_0^{\pi} (f(x) + f''(x))\sin x\ dx\ =\ 5\ \Rightarrow\ 2 + f(0) = 5\ \Rightarrow\ \fbox{$f(0) = 3$}$

Integral do arco-tangente.

$\displaystyle\int \arctan x\ dx\ \overset{\text{Por partes}}=\ x \cdot \arctan x - \underset{I}{\underbrace{\displaystyle\int \dfrac{x}{x^2 + 1} dx}}$

Seja $u = x^2 + 1$, $du = 2x\ dx$.

$I = \dfrac{1}{2} \displaystyle\int \dfrac{du}{u} = \dfrac{\log |u|}{2} + c = \dfrac{\log |x^2 + 1|}{2} + c = \dfrac{\log (x^2 + 1)}{2} + c$

Logo, $\fbox{$\displaystyle\int \arctan x\ dx\ =\ x \cdot \arctan x - \dfrac{\log (x^2 + 1)}{2} + C$}$.